Question

The matrix A=[-17-51-85-21 is row equivalent to R=「1 3 5 15 45 75 1 -4 -12 -20 0 1. a. Find a basis for the row space of A, r

0 0
Add a comment Improve this question Transcribed image text
Answer #1

Nr 1 35 0 ? (-r+,-51,-85,-リ자 1H2 A 3 AX 3-53 거9 t0

Add a comment
Know the answer?
Add Answer to:
The matrix A=[-17-51-85-21 is row equivalent to R=「1 3 5 15 45 75 1 -4 -12...
Your Answer:

Post as a guest

Your Name:

What's your source?

Earn Coins

Coins can be redeemed for fabulous gifts.

Not the answer you're looking for? Ask your own homework help question. Our experts will answer your question WITHIN MINUTES for Free.
Similar Homework Help Questions
  • Consider the matrix 0 4 8 24 0-3-6 3 18 A-0 24 2 -12 0 -2-3...

    Consider the matrix 0 4 8 24 0-3-6 3 18 A-0 24 2 -12 0 -2-3 0 7 0 3 5 [51 [51 a) Find a basis for the row space Row(A) of A (b) Find a basis for the column space Col(A) of A (c) Find a basis space d) Find the rank Rank(A) and the nullity of A (e) Determine if the vector v (1,4,-2,5,2) belongs to the null space of A. - As always,[5 is for the...

  • 1. Consider the matrix 12 3 4 A 2 3 4 5 3 4 5 6...

    1. Consider the matrix 12 3 4 A 2 3 4 5 3 4 5 6 As a linear transformation, A maps R' to R3. Find a basis for Null(A), the null space of A, and find a basis for Col(A), the column space of A. Describe these spaces geometrically. 2. For A in problem 1, what is Rank(A)?

  • Given A and b to the​ right, write the augmented matrix for the linear system that corresponds to the matrix equation Axequals=b. Then solve the system and write the solution as a vector. Aequals=left...

    Given A and b to the​ right, write the augmented matrix for the linear system that corresponds to the matrix equation Axequals=b. Then solve the system and write the solution as a vector. Aequals=left bracket Start 3 By 3 Matrix 1st Row 1st Column 1 2nd Column 3 3rd Column negative 2 2nd Row 1st Column negative 2 2nd Column negative 2 3rd Column 0 3rd Row 1st Column 5 2nd Column 2 3rd Column 6 EndMatrix right bracket 1...

  • 1. Consider the following matrix and its reduced row echelon form [1 0 3 3 5...

    1. Consider the following matrix and its reduced row echelon form [1 0 3 3 5 187 [1 0 3 3 0 37 1 1 5 4 1 10 0 1 2 1 0 - A=1 4 1 0 3 3 -1 0 rref(A) = 10 0 0 0 1 3 2 0 6 6 -1 3 | 0 0 0 0 0 0 (a) Find a basis of row(A), the row space of A. (b) What is the dimension...

  • Find a basis for the row space of A. 1 -1 3 2 -3 8 A-0...

    Find a basis for the row space of A. 1 -1 3 2 -3 8 A-0 1 -2 Find a basis for the null space of A. Verify that every vector in row(A) is orthogonal to every vector in null(A). Need Help? Submit Answer Save Progress Practice Another Version 17. -12 points PooleLinAlg4 5.2.009. Find a basis for the column space of A. My Notes Ask Your Tea 1-1 3 5 2 1 A- 012 T. Find a basis for...

  • 1 — 0 1 1 [R |d 1 Consider the augmented matrix [A | b) and...

    1 — 0 1 1 [R |d 1 Consider the augmented matrix [A | b) and its reduced row echelon form [Ra]: 2 -2 0 23 6 0 4 0 7 / 4 -1 -1 0-15 | -5 row operations -3 0 [ A ] b] = 81 -2 -4 4 -35-10 0 0 0 11 12 3 6 -60 69 18 0 0 0 0 0 1 0 (a) Write the vector form of the general solution to the...

  • 2. Let A be the matrix [i 3 4 51 0 A= 1 1 1 |...

    2. Let A be the matrix [i 3 4 51 0 A= 1 1 1 | 1 2 -4 -5 -3 -3 -2 -1 (a) Find a basis of the column space. Find the coordinates of the dependent columns relative to this basis. (b) What is the rank of A? (c) Use the calculations in part (a) to find a basis for the row space.

  • For the following matrix: [1 1 2] |1 1 2| = A [2 3 5] a)...

    For the following matrix: [1 1 2] |1 1 2| = A [2 3 5] a) Find a matrix B in reduced echelon form such that B is row equivalent to the given matrix A. b) Find a basis for the nullspace of A. c) Find a basis for the range of A that consists of columns of A. For each column, Aj, of A that does not appear in the basis, express Aj as a linear combination of the...

  • Question 3 please answer clearly. A matrix A and its reduced row echelon form are given...

    Question 3 please answer clearly. A matrix A and its reduced row echelon form are given as follows: [ 2 1 3 41 | 1 2 0 2 A= 3 21 12 | 3 -1 7 9 18 7 9 -4 and rref(A) = [ 1 0 201 0 1 -1 0 0 0 0 1 0 0 0 0 | 0 0 0 0 Use this information to answer the following questions. (a) Is the column vector u= in...

  • eclass.srv.ualberta.ca 2 of 2 1. Consider the matrix 3-2 1 4-1 2 3 5 7 8...

    eclass.srv.ualberta.ca 2 of 2 1. Consider the matrix 3-2 1 4-1 2 3 5 7 8 (a) Find a basis B for the null space of A. Hint: you need to verify that the vectors you propose 20 actually form a basis for the null space. (Recall: (1) the null space of A consists of all x e R with Ax = 0, and (2) the matrix equation Ax = 0 is equivalent to a certain system of linear equations.)...

ADVERTISEMENT
Free Homework Help App
Download From Google Play
Scan Your Homework
to Get Instant Free Answers
Need Online Homework Help?
Ask a Question
Get Answers For Free
Most questions answered within 3 hours.
ADVERTISEMENT
ADVERTISEMENT
ADVERTISEMENT